Which type of function best models the given data?

Which Type Of Function Best Models The Given Data?

Answers

Answer 1

Answer:

3)

Step-by-step explanation:

Draw the graph and youll see

Answer 2

Answer:

Exponential growth function

Step-by-step explanation:

It multiplies by 3 each time


Related Questions

what is the difference of the polynomials?
{m^2n^2-7}-(mn+4)

Answers

Answer:

[tex]m^{2} n^{2} - mn-11[/tex]

Step-by-step explanation:

Simplify the expression.

How do you find co terminal angle of -445

Answers

Answer:

275°

Step-by-step explanation:

-445+360=-85+360=275°

an estate valued at $75 000 is divided among three daughters Natasha, Natalie and Nadia in the ratio 5:8:2 respectively. Calculate the amount each receives ​

Answers

Answer:

Natasha: $25000

Natalie: $40000

Nadia: $10000

Step-by-step explanation:

5 : 8 : 2

5 + 8 + 2 = 15

$75000 ÷ 15

$5000

$5000 × 5 = $25000

$5000 × 8 = $40000

$5000 × 2 = $10000

What is the answer to 4/5 of 30?

Answers

Answer:

24

Step-by-step explanation:

[4 / 5 ] * 30 = 24

Answer:

24

Step-by-step explanation:

To get the answer you have to multiply the 4/5 x 30 to get the answer

Which sentence explains why polygon ABCDEFGH is congruent to polygon MNOPQRST?


A.
A sequence of translations will map MNOPQRST onto ABCDEFGH.
B.
A sequence of rigid transformations will map MNOPQRST onto ABCDEFGH.
C.
A sequence of rotations will map MNOPQRST onto ABCDEFGH.
D.
A sequence of reflections will map MNOPQRST onto ABCDEFGH.

Answers

Answer:

correct option if Option D

Answer:

The answer is B.

Step-by-step explanation:

It can't be A, C, or, D because if the polygons were in a certain position, those transformations wouldn't work.

The definition of a rigid transformation is either a translation, rotation, or reflection. Therefore it must be B.

Sorry about how vague this is. It's hard to explain this without diagrams.

In a survey, 15 high school students said they could drive and 15 said they could not. Out of 60 college students surveyed, 30 said they could drive. Micah concluded that knowing that a person is in college means they are more likely to drive. Is Micah’s conclusion correct? Explain.

Answers

Answer:

No.

Step-by-step explanation:

#1, the sample size is not indicative of a proper high school population, unless of all there is a small amount of students in the high school (or even only 30 students in it). On a individual basis, it can vary widely, and so data analysis such as this is not correct. The data size is skewed as well for the college, especially as college campuses typically house more students then usual. To only ask 60 college students also skew the data gotten. Another sample error may be locational error, in which the location of the schools may warrant the need for a driver license, or if it is not needed, as well as the distance of the student's house and/or where needed facilities are to accommodate a student's needs and desires.

If we were to set all these problems aside and focus more on the question, 15 high school students said they could drive, and 15 said they could not, which makes the sample size of 30 high school students in all. 15 say they can drive, therefore:

15/30 = 1/2, or there is a 50% chance of obtaining a student that can drive.

On the other hand, out of 60 college students surveyed, 30 said they can drive, therefore:

30/60 = 1/2, or there is a 50% chance of obtaining a student that can drive.

∴ Micah's prediction is incorrect in obtaining a college student that can drive more easily then obtaining one that can drive in a high school.

However, remember that too small of a data size can lead to a heavy skew, and it will not represent reality as well.

~

Answer:

Sample Response: No, Micah is not correct. Since there are equal numbers in the group that drive and the group that doesn’t drive, the relative frequency for each is 50%. Because the relative frequencies are the same, there is no association between the variables.

Step-by-step explanation:

Got this from Edg 2021

slope = 4, point (3,-2) is on the line

Answers

Answer:

y = 4x + b

Step-by-step explanation:

We know that (3, -2) is on this line. So 3 = m(-2)+b.

This leaves us with two variables the slope (m) and the y intercept (b).

But wait a minute, the slope is given to us since it says slope = 4.

Our equation is now, 3 = 4(-2)+b

3 = -8 + b

b = 11.

Our final equation is now y = 4x + b.

(y,x) is a point on this line.

Hope this helped!

JoeLouis2

Find the percent of markup on a T-shirt that has a store cost of $4.87 and a selling price of $15.95

Answers

Answer:

69.47%

Step-by-step explanation:

Calculation to determine Find the percent of markup

Using this formula

Percent of markup=New price-Old price/Old price

Let plug in the formula

Percent of markup=$15.95-$4.87/$15.95

Percent of markup=$11.08/$15.95

Percent of markup=0.6947*100

Percent of markup=69.47%

Therefore the percent of markup is 69.47%

9. A sector of a circle has central angle
[tex] \frac{\pi }{4} [/tex]
and area
[tex] \frac{49\pi}{8} [/tex]

[tex] {cm}^{2} [/tex]
. Find the radius of the circle​

Answers

Answer:

Step-by-step explanation:

Begin with the formula for the area of a sector:

[tex]A=\frac{\theta}{2\pi}*\pi r^2[/tex] and we are given everything except the radius, r. Filling in what we know:

[tex]\frac{49\pi}{8}=\frac{\frac{\pi}{4} }{2\pi}*\pi r^2[/tex] and then start the simplification process. I began by dealing with the fraction over a fraction thing, which simplifies the equation to:

[tex]\frac{49\pi}{8}=\frac{1}{8}\pi r^2[/tex] then isolate the r-squared:

[tex]\frac{8(49\pi)}{8\pi}=r^2[/tex]. Canceling out the π's and the 8's leaves us with simply

r² = 49 so

r = 7

suppose y varies inversely with x, and y=39 when x=1/3.what is the value of y when x=26

Answers

Answer:

y = 1/2

Step-by-step explanation:

Varies inversely

xy = k

------------------------

Find the constant of variation

y = 39 when x = 1/3

k = 1/3 * 39

k = 13

-----------------------

what is the value of y when x=26

26y = 13

y = 13/26

y = 1/2

Answer:

y = [tex]\frac{1}{2}[/tex]

Step-by-step explanation:

Given y varies inversely with x then the equation relating them is

y = [tex]\frac{k}{x}[/tex] ← k is the constant of variation

To find k use the condition y = 39 when x = [tex]\frac{1}{3}[/tex] , then

39 = [tex]\frac{k}{\frac{1}{3} }[/tex] ( multiply both sides by [tex]\frac{1}{3}[/tex] )

[tex]\frac{1}{3}[/tex] × 39 = k

13 = k

y = [tex]\frac{13}{x}[/tex] ← equation of variation

When x = 26 , then

y = [tex]\frac{13}{26}[/tex] = [tex]\frac{1}{2}[/tex]

How many x-intercepts appear on the graph of this polynomial function?
f(x) = x - 5x?
O 1 x-intercept
O 2 x-intercepts
O 3 x-intercepts
O 4 x-intercepts

Answers

Answer: B

Step-by-step explanation:

There are two x-intercepts: x = -√5 and x = √5. The answer is: 2 x-intercepts.

What is x-Intercept?

The x-intercept is defined as an intercept that is located at the x-axis of the plane, is the location or coordinate from where the line crosses.

To find the x-intercepts of the polynomial function f(x) = x⁴ - 5x², we set f(x) equal to zero and solve for x.

Setting f(x) = 0:

x⁴ - 5x² = 0

Factoring out the common term x²:

x²(x² - 5) = 0

Now we have two factors: x² = 0 and x² - 5 = 0.

For x² = 0, we have a double root at x = 0.

This means the graph touches the x-axis at x = 0 but does not cross it.

For x² - 5 = 0, we can solve for x by taking the square root:

x² = 5

x = ±√5

Therefore, there are two x-intercepts: x = -√5 and x = √5.

The answer is: 2 x-intercepts.

Learn more about the x-intercept here:

https://brainly.com/question/14180189

#SPJ7

Which equation shows the point-slope form of the line that passes through (3, 2) and has a slope of 1/3

Answers

Answer:

y-y1=m(x-x1)

y-2=1/3(x-3)

y=1/3x+1

What is the volume of this. Pyramid

720 cm2
1080cm2
1440 cm2
2160 cm2

Answers

Answer:

Option (1)

Step-by-step explanation:

To calculate the volume of the given pyramid,

Formula to be used,

Volume = [tex]\frac{1}{3}(\text{Area of the triangular base)}(\text{Height})[/tex]

Area of the triangular base = [tex]\frac{1}{2}(\text{Base})(\text{Height})[/tex]

                                             = [tex]\frac{1}{2}(9)(15)[/tex]

                                             = 67.5 cm²

Now volume of the pyramid = [tex]\frac{1}{3}(67.5)(32)[/tex]

                                               = 720 cm³

Therefore, Option (1) will be the correct option.

A box shown below is to paint blue except the bottom. What is the area of the surface, and square inches, that will be covered with the paint?

Answers

Answer:

186

Step-by-step explanation:

Your entire surface area is

216

the area of the rectangle on the bottom is 30

substract 30 from 216

the answer is 186 cause u subtract 30 from 216

Ranking brainliest if correct :)))

Answers

Answer:

Hello! answer: always

Step-by-step explanation:

And Isosceles triangle is a triangle with two sides that have the same length therefore the answer is always because as shown there are two sides with the same length hope that helps!

Answer is: always

since a  isosceles is a triangle with two equal sides and since two of the number is same(you know what i mean)


A whole number is 6 more than 2 times another number. The sum of the two numbers is less than 50. This can be written in an inequality as x+2x+6<50, where xrepresents the smaller number.
From the set (13, 14, 15, 16, 17), the values of x for which the inequality holds true are

Answers

Answer:

13, 14

Step-by-step explanation:

The parameters of the numbers are;

A whole number value = 2 × Another number + 6

The sum of the two numbers is less than 50

Given that the first number is equal to more than twice the second number, we have that the first number is the larger number, while the second number is the smaller number

Where 'x' represents the second number, we get;

x + 2·x + 6 < 50

Simplifying gives;

3·x + 6 < 50

x < (50 - 6)/3 = 14.[tex]\overline 6[/tex]

x < 14.[tex]\overline 6[/tex]

Therefore, the numbers for which the inequality holds true are numbers less than 14.[tex]\overline 6[/tex]. From the given option, the numbers are 13, and 14.

Out of the people who have already taken their seats at a seminar, 2 people have black hair while 2 people do not. Considering this data, how many of the next 16 people to take their seats should you expect to be black-haired?

Answers

Answer:8

Step-by-step explanation:

We can expect 8 of the next 16 people to have black hair, based on the given information.

What is Probability?

It is a branch of mathematics that deals with the occurrence of a random event.

Since 2 out of the total number of people who have already taken their seats at the seminar have black hair

we can assume that the probability of a person having black hair is 2 out of the total number of people who have taken their seats.

The probability of a person having black hair can be expressed as:

P(black hair) = 2/(2+2) = 1/2

This means that out of the total number of people who have already taken their seats, half of them have black hair.

If we assume that the next 16 people to take their seats are chosen randomly and independently from the population,

we can use the expected value formula to calculate the number of people with black hair we would expect among those 16 people:

Expected value = total number of people x probability of having black hair

Expected value = 16 x (1/2)

Expected value = 8

Therefore, we can expect 8 of the next 16 people to have black hair, based on the given information.

To learn more on probability click:

https://brainly.com/question/11234923

#SPJ2

Q7) A rectangular piece of land measures 8 m by 5m. It is to be fenced with four lines of wire. What length of wire is needed for fencing?

Answers

Answer:

26 m

Step-by-step explanation:

Find what length is needed for fencing by finding the perimeter of the land.

Use the perimeter formula, P = 2l + 2w

Plug in the length and width:

P = 2l + 2w

P = 2(8) + 2(5)

P = 16 + 10

= 26

So, the length of wire needed is 26 m

The growth rate of some species of bamboo is 0.0000237 miles per hour. write this in scientific notation

Answers

Answer:

2.37 x 10 ^ -5

Step-by-step explanation:

What are the solutions of x2 - 2x +17 - 0?
O A. x = 1+2, or x = 1 - 2
O B. X = 1+4, or x = 1- 47
O c. x = 2, or x = -2
O D. x = 4; or x = -4/

Answers

Answer:

option B

Step-by-step explanation:

[tex]x^2 - 2x + 17 = 0 \ , \ [ \ where \ a = 1 , \ b = - 2 \ c = 17 \ ][/tex]

[tex]x = \frac{- b \ \pm \sqrt{b^2 - 4ac}}{2a}\\\\x = \frac{2 \ \pm \sqrt{2^2 - (4 \times 1 \times 17)}}{ 2 \times 1}\\\\x = \frac{2 \ \pm \sqrt{4 - 68}}{ 2 }\\\\x = \frac{2 \ \pm \sqrt{-64}}{ 2 }\\\\x = \frac{2 \ \pm \sqrt{8^2 \times -1}}{ 2 }\\\\x = \frac{2 \ \pm \ 8\sqrt{ -1}}{ 2 }\\\\[/tex]              

[tex]x = \frac{2 \ \pm\ 8 i}{ 2 }\\\\[/tex]                               [tex][ \ where \ i = \ \sqrt{i} \ ][/tex]

[tex]x = 1 \ \pm \ 4i\\\\x = 1 + 4i , \ x = 1 - 4i[/tex]

Answer:

O B.  [tex]x=1+4i[/tex] or [tex]x=1-4i[/tex]

Step-by-step explanation:

Use the Quadratic Formula for the Quadratic Equation!

Quadratic Equation: [tex]x^{2} -2x+17=0[/tex]

Quadratic Formula: [tex]x= \frac{-b±\sqrt{b^2-4ac} }{2a}[/tex]

**********************************************************************

Now set up the values like this!

[tex]ax^{2} +bx+c[/tex]

**********************************************************************

[tex]x=\frac{-b±\sqrt{-2^2-(4(0)(17)} }{2(0)}[/tex]

[tex]x=\frac{-b±\sqrt{4-(1)} }{0}[/tex]

x = 1 - 4i

x = 1 + 4i

In the applet above right triangle XYZ is drawn with a 23 degrees angle. for the right triangles with an angle of 23 degrees the ratio of the leg adjacent the 23 degrees angle compared to the hypotenuse is 0.92 if the leg adjacent to the 23 degrees angle has a length of 46 units, what is the length of the hypotenuse ?

Answers

Answer:

50 ( nearest whole number)

Step-by-step explanation:

Given from the question :

Ratio of Adjacent to hypotenus = 0.92

Length of Adjacent leg = 46 units

From trigonometry :

Ratio of adjacent to hypotenus is the cosine of the angle :

Hence,

Cos 23 = adjacent / hypotenus

Cos 23 = 46 / hypotenus

0.9205048 = 46/ hypotenus

Hypotenus = 46 / 0.9205048

Hypotenus = 49.97

Please help :( The picutre is right below worth 20 pionts and will mark brainliest :)
oh and the question is right in the picture too also with the rest

Answers

Answer:

perpendicular

Step-by-step explanation:

peripenducular looks like a plus sign. HOPE THIS HELPS!

Branliest...

Anthony and Joan have $10 to buy apples and oranges at the market. They will buy a total
of 25 pieces of fruit. Apples cost 50 cents each and oranges cost 25 cents each. Which of
these systems best represents this situation?

Answers

The last one

x + y = 25 is the number of pieces of fruit they will buy, where x is apples and y is oranges.

0.50x + 0.25y = 10 applies the cost, in cents, to each piece of fruit. The 10 is 10.00, keeping everything to the same number of decimal places.

Find the missing side of the triangle. Round your answers to the nearest tenth if necessary.



15.3 m

10 m

24.6 m

22.4 m

Answers

Answer:

Solution given:

perpendicular[p]=x

base[b]=11.6m

hypotenuse[h]=19.2m

by using Pythagoras law

h²=p²+b²

19.2²-11.6²=x²

x=[tex]\sqrt{234.08}[/tex]

x=15.3m

option

a.

15.3 m

Answer:

15.3 m

Step-by-step explanation:

c refers to the longest side (hypotenuse) whereas a and b referes to the two other smaller sides of atriangle.

pythagoras theorem

a^2 + b^2 = c^2

11.6^2 + x^2 = 19.2^2

134.56 + x^2 = 368.64

x^2 = 368.64 - 134.56

x^2 = 234.08

x=[tex]\sqrt{234.08[/tex]

x=15.29

x=15.3 m


[tex]\huge\mathfrak\red{please \: solve \: } \\ \\ {\huge{\underline{\bf{\green{factorise: - }}}}} \\ \\ {\huge{\boxed{\sf{\purple{2x {}^{2} + 9x + 10 }}}}}[/tex]

Answers

Answer:

(x + 2)(2x + 5)

Step-by-step explanation:

2x^2 + 9x + 10 =

This is a trinomial of the form ax^2 + bx + c, where a is not 1.

Step 1: Try to factor out a common factor.

In this case, there is no common factor in all terms.

We will now factor by grouping.

Step 2. Find the product ac.

ac = 2 * 10 = 20

Step 3. Find two factors of ac whose product is ac and whose sum is b.

In this case, we need two numbers whose product is 20 and whose sum is 9.

Those numbers are 4 and 5.

Step 4. Break up the middle term using the two numbers we just found.

9x becomes 4x + 5x

Now we have

= 2x^2 + 4x + 5x + 10

Step 5. Factor by grouping. Factor a common factor out of the first two terms. Factor a common factor out of the last two terms.

= 2x(x + 2) + 5(x + 2)

Finally, factor out the common factor, x + 2.

= (x + 2)(2x + 5)

Answer: (x + 2)(2x + 5)

ans thissss plsssssssassss

Answers

Answer:

21.16 m^2

Step-by-step explanation:

perimeter of square=4l

18.4/4=l

4.6=l

area of square=l^2

=4.6^2

=21.16 m^2

Please help me solve my problem!!!

Answers

True

Hope this helps! :)

______________

Find the sum of the numbers. Express your answer in scientific notation.
(6.94 x 10-7)+(2.7x10-)
O 7.21x 10-
O 9.64 x 10-
O 7.21x10-2
O 9.64 x 10-7

Answers

Answer:

7.21 × 10⁻⁷

Step-by-step explanation:

(6.94 × 10⁻⁷) + (2.7 × 10⁻⁸) = 7.21 × 10⁻⁷

PLZ HELP WILL GIVE BRAINLIEST
A company wants to provide better customer service to all of its 2000 clients. The company needs to create a sample of clients to survey.

How should the company select its sample of clients? Select two that apply.

The company should choose the first 50 clients that volunteer to take the survey.
The company should have a computer randomly generate a list of 200 clients to survey.
The company should survey every 5th client that makes an appointment.
Every employee in the company should recommend 1 client to survey.

Answers

Answer:

the company should survey every 5th client that makes an appointment.

the company should have a computer randomly generate a list of 200 clients to survey.

This is correct? I need help please :(

Answers

Answer:

well, we only no that

2l + 2w is 80

(lengths and widths on all 4 sides

and that w * l is A

so the function should be w times what's left for l, but also expressed by something with w

2l + 2w = 80 | -2w

2l = 80 -2w | devide by 2

l = 80 - w

now we can substitute l in

A = w * l

so that we only need w's

A(w) = w * (80-w)

for any w it will give us the area, makes only sense for 0<w<80

ye that’s right i think

-
a
a
a
a
a
a

a
a
a
a
a
a
Other Questions
Write a program that reads a string and outputs the number of times each lowercase vowel appears in it. Your program must contain a function with one of its parameters as a string variable and return the number of times each lowercase vowel appears in it. Also write a program to test your function. (Note that if str is a variable of type string, then str.at(i) returns the character at the ith position. The position of the first character is 0. Also, str.length() returns the length of the str, that is, the number of characters in str.) La liga de los pelirrojos que lugares se encuentran a la vuelta de la calzada del negocio de wilson en saxe carburgo squere What type of religious freedom existed in the Soviet Union? one religion many religions complete none . - ? t Anne is painting her house light blue. To make the color she wants, she must add 3 cans of white paint to every 2 cans of blue paint. How many cans of white paint will she need to mix with 6 cans of blue? what does ruby bridges movie and book have different why did ralph waldo emerson call the colonists indians? What impact did railroads have on cities across the United States at the turn of the 20th century? PLEASE HELP ME WITH THIS ONE QUESTION The half-life of Barium-139 is 4.96 x 10^3 seconds. A sample contains 3.21 x 10^17 nuclei. How much of the sample is left after 1.98 x 10^4 seconds?A) 8.03 x 10^16 nucleiB) 4.01 x 10^16 nucleiC) 2.02 x 10^16 nucleiD) 1.61 x 10^17 nuclei 50 POINTS + BRAINLIEST ________ protects our organs, insulates our bodies, and help carry vitamins A, D, E, and K. Fat Protein Sugar Water When the mean value of the dependent variable is independent of variation in the independent variable, the slope of the regression line is:________ please answer this question Please help find the volume by using 3.14 as pi PLS HELP!!! The function c = 1.50(n 2) + 5.50 represents the cost c in dollars of printing n invitations. Which of the following is not true? A. Each invitation costs $5.50 to print. B. For each additional invitation, it costs an extra $1.50 to print. C. One can not print just one invitation. D. The cost depends on the number of invitations printed. In his backyard, a homeowner unwittingly built a garage that encroached two feet across the property line onto property owned by his neighbor. The next year, the homeowner discovered his error in the course of selling his property to a buyer. After the sale, the neighbor learned of the encroachment of the garage onto her property and sued the homeowner for damages and trespass. In this action, will the neighbor prevail Define malnutrition and identify a specific disease that can result from it. HELPPP!!! PLEASEEEEEEE Reflection: Across the line y = -1 Help me please. NO LINKS Please help thanks this is due today!